Finding Eigenvalues and Eigenfunctions for O.D.E. Problem on Interval [0, 4π]

Click For Summary
The discussion focuses on solving the eigenvalue problem defined by the differential equation f'' + λf = 0 on the interval [0, 4π] with specific boundary conditions f'(0) = 0 and f'(4π) = 0. Participants clarify that the task is to find all eigenvalues (λ) and corresponding eigenfunctions (f(x)) while considering only non-negative values for λ. There is a suggestion to explore negative values for λ as extra credit, indicating that this approach leads to only the trivial solution. The conversation includes a request for feedback on the correctness of the attempted solution, highlighting the collaborative nature of the inquiry. Overall, the thread emphasizes the mathematical process of identifying eigenvalues and eigenfunctions in the context of boundary value problems.
jegues
Messages
1,085
Reaction score
3

Homework Statement



Solve the differential equation eigenvalue problem:

f'' + \lambda f = 0, \quad 0 \leq x \leq 4\pi, \quad \text{where} \quad f^{'}(0) =0, \quad f^{'}(4\pi) = 0, \quad \text{and} f \neq 0.

Consider ONLY \quad \lambda \geq 0, \quad and find the values of \quad \lambda \quad and f(x).

Homework Equations





The Attempt at a Solution



See figure attached for my attempt at the solution

By "Solve the differential equation eigenvalue problem" do they simply mean find all the eigen values and eigen functions?

If so, is what I've done correct?

Thanks again!
 

Attachments

  • 2006Q2.jpg
    2006Q2.jpg
    28.1 KB · Views: 384
Physics news on Phys.org
It looks OK to me.

For extra credit, instead of just assuming λ ≥ 0, try λ = -μ2 < 0 and show you only get the trivial solution.:cool:
 
Question: A clock's minute hand has length 4 and its hour hand has length 3. What is the distance between the tips at the moment when it is increasing most rapidly?(Putnam Exam Question) Answer: Making assumption that both the hands moves at constant angular velocities, the answer is ## \sqrt{7} .## But don't you think this assumption is somewhat doubtful and wrong?

Similar threads

Replies
4
Views
2K
Replies
5
Views
2K
  • · Replies 7 ·
Replies
7
Views
1K
Replies
41
Views
5K
  • · Replies 1 ·
Replies
1
Views
2K
  • · Replies 9 ·
Replies
9
Views
3K
  • · Replies 6 ·
Replies
6
Views
2K
  • · Replies 2 ·
Replies
2
Views
2K
  • · Replies 2 ·
Replies
2
Views
1K
  • · Replies 1 ·
Replies
1
Views
2K